Forgot password?
 Create new account
View 331|Reply 13

[不等式] 若 a>b>c>0,a+b+c=6,证明 (a - b) (a - c) (b - c) a b c ≤ 27。

[Copy link]

123

Threads

463

Posts

3299

Credits

Credits
3299

Show all posts

TSC999 Posted at 2025-3-6 19:34:39 |Read mode
若 \(a > b > c > 0\),\(a + b + c = 6\),证明 \(f(a,b,c) = (a - b) (a - c) (b - c) a b c ≤27\)。

当 \(a、b、c\) 各取何值时 \(f(a,b,c)\) 取得最大值?

701

Threads

110K

Posts

910K

Credits

Credits
94172
QQ

Show all posts

kuing Posted at 2025-3-6 19:40:26

123

Threads

463

Posts

3299

Credits

Credits
3299

Show all posts

 Author| TSC999 Posted at 2025-3-7 08:33:31
Last edited by hbghlyj at 2025-3-7 16:09:08
Notice that the left of the inequality is shift invariant, so Wlog $d=0$.
Then the inequality becomes $a+b+c=6$,prove
$a b c(a-b)(a-c)(b-c) \leq 27$.
Set $m=4 \cos ^2 10, n=4 \cos ^2 50, p=4 \cos ^2 70$. It's easy to get
$$
\frac{1}{m}+\frac{1}{m-n}+\frac{1}{m-p}=\frac{1}{n}+\frac{1}{n-p}+\frac{1}{n-m}=\frac{1}{p}+\frac{1}{p-m}+\frac{1}{p-n}=1
$$
and
$$
(m-n)(m-p)(n-p)=9, m n p=3
$$

Thus by AM-GM,
$$
\begin{aligned}
& a b c(a-b)(a-c)(b-c) \\
& =m n p(m-n)(m-p)(n-p) \frac{a}{m} \frac{b}{n} \frac{c}{p} \frac{a-b}{m-n} \frac{a-c}{m-p} \frac{b-c}{n-p} \\
& =27 \frac{a}{m} \frac{b}{n} \frac{c}{p} \frac{a-b}{m-n} \frac{a-c}{m-p} \frac{b-c}{n-p} \\
& \leq 27\left(\frac{\frac{a}{m}+\frac{b}{n}+\frac{c}{p}+\frac{a-b}{m-n}+\frac{a-c}{m-p}+\frac{b-c}{n-p}}{6}\right)^6 \\
& =27\left(\frac{\sum\left(\frac{1}{m}+\frac{1}{m-n}+\frac{1}{m-p}\right) a}{6}\right)^6 \\
& =27\left(\frac{a+b+c}{6}\right)^6 \\
& =27
\end{aligned}
$$
上面这个来自【知乎】的证明没看懂,这个证明也没有给出取得最大值时的 \(a,b,c\) 值。
答案是 \(f(a,b,c)\) 有最大值 \(27\) 时,\(a,b,c\) 是一元三次方程 \(x^3-6 x^2+9 x - 3=0\) 的三个根。即
\(a=4 cos^2\frac{π}{18}\),\(b=4 cos^2\frac{5π}{18}\),\(c=4 cos^2\frac{7π}{18}\)。

123

Threads

463

Posts

3299

Credits

Credits
3299

Show all posts

 Author| TSC999 Posted at 2025-3-7 09:43:19
此题来源,据抖音上张文龙老师说,是韦神(韦东奕)17 岁时由于在国家集训队表现出色,教练为他量身定制的一道题。据说当年韦东奕轻松的解出了这道题。
张文龙在抖音上的解法是用拉格朗日乘子法。最终是归结为解一个一元三次方程 \(x^3-6 x^2+9 x-3=0\),这一步我没有看懂。关键是他推出了以下关系式:
\(\frac{1}{a}+\frac{1}{b}+\frac{1}{c}=3\) 和 \(a^2+b^2+c^2=18\),结合已知条件 \(a+b+c=6\),由此可推知 \(a b+b c+c a=9\) 以及 \(a b c =3\),从而
\(a,b,c\) 是一元三次方程 \(x^3-6 x^2+9 x-3=0\)的三个根。

找张文龙老师在抖音上的视频,进入抖音后,搜索抖音号 882497791,进入张文龙的主页后,再搜索【数学的江湖】,就是这个视频。

701

Threads

110K

Posts

910K

Credits

Credits
94172
QQ

Show all posts

kuing Posted at 2025-3-7 18:16:21
Last edited by kuing at 2025-3-7 22:52:53
TSC999 发表于 2025-3-7 08:33
上面这个来自【知乎】的证明没看懂,这个证明也没有给出取得最大值时的 \(a,b,c\) 值。
答案是 \(f(a, ...
该截图是截自 artofproblemsolving.com/community/c6h432851p8689589 而不是知乎。

这其实是一个待定系数均值,待定 `t_i>0`, `i\in\{1,\dots,6\}`,由均值有
\begin{align*}
&abc(a-b)(a-c)(b-c)\\
={}&t_1t_2t_3t_4t_5t_6\frac a{t_1}\frac b{t_2}\frac c{t_3}\frac{a-b}{t_4}\frac{a-c}{t_5}\frac{b-c}{t_6}\\
\leqslant{}&\frac{t_1t_2t_3t_4t_5t_6}{6^6}\left(\frac a{t_1}+\frac b{t_2}+\frac c{t_3}+\frac{a-b}{t_4}+\frac{a-c}{t_5}+\frac{b-c}{t_6}\right)^6\\
={}&\frac{t_1t_2t_3t_4t_5t_6}{6^6}\left(\left(\frac1{t_1}+\frac1{t_4}+\frac1{t_5}\right)a+\left(\frac1{t_2}-\frac1{t_4}+\frac1{t_6}\right)b+\left(\frac1{t_3}-\frac1{t_5}-\frac1{t_6}\right)c\right)^6,
\end{align*}
那么取最值时上式应能化为 `(\cdots)(a+b+c)^6` 的形式,因此需要
\[\frac1{t_1}+\frac1{t_4}+\frac1{t_5}=\frac1{t_2}-\frac1{t_4}+\frac1{t_6}=\frac1{t_3}-\frac1{t_5}-\frac1{t_6},\]
同时还得满足均值那一步的取等,即
\[\frac a{t_1}=\frac b{t_2}=\frac c{t_3}=\frac{a-b}{t_4}=\frac{a-c}{t_5}=\frac{b-c}{t_6},\]
综合以上两式可知,取最值时的 `a`, `b`, `c` 应满足
\[\frac1a+\frac1{a-b}+\frac1{a-c}=\frac1b-\frac1{a-b}+\frac1{b-c}=\frac1c-\frac1{a-c}-\frac1{b-c},\]
结合已知条件,记
\[g(a,b,c)=\frac1a+\frac1{a-b}+\frac1{a-c},\]
即最值时的 `a`, `b`, `c` 满足 `a+b+c=6` 且
\[g(a,b,c)=g(b,c,a)=g(c,a,b),\]
这样就可以解出 `a`, `b`, `c`。

关键就是如何解这个方程组,我没什么好的想法,或许会有妙解,我没抖音,也看不到楼上说的那个推导。

不过不要紧,我们有软件,暴力算总是可以的,计算得
\begin{align*}
&g(a,b,c)=g(b,c,a)\\
\riff{}& 2a^3b-6a^2b^2+2ab^3-a^3c+3a^2bc+3ab^2c-b^3c+a^2c^2-4abc^2+b^2c^2=0,
\end{align*}
于是再令 `h(a,b,c)=2a^3b-6a^2b^2+2ab^3-a^3c+3a^2bc+3ab^2c-b^3c+a^2c^2-4abc^2+b^2c^2`,即有
\[h(a,b,c)=h(b,c,a)=0,\]
此时代入 `c=6-a-b`,再由这两式消去 `b`,即可得到 `a` 满足的方程,在 MMA 上输入以下命令
  1. h[a_, b_, c_] = 2 a^3 b - 6 a^2 b^2 + 2 a b^3 - a^3 c + 3 a^2 b c + 3 a b^2 c - b^3 c + a^2 c^2 - 4 a b c^2 + b^2 c^2;
  2. Resultant[h[a, b, 6 - a - b], h[b, 6 - a - b, a], b] // Factor
Copy the Code

即可得到
\[-15116544 (-6 + a)^2 (-3 + a) (-2 + a)^4 a^3 (-3 + 9 a - 6 a^2 + a^3)^2=0,\]
其中 `a=2`, `3`, `6` 会使得原式分母为零而舍去,这样就得到 `-3 + 9 a - 6 a^2 + a^3=0`,也就是取等时的 `a`, `b`, `c` 是方程 `x^3-6x^2+9x-3=0` 的三个根。

在 `x^3-6x^2+9x-3=0` 中令 `x=2+2\cos\theta`,即化为 `2\cos3\theta-1=0`,所以 `\theta=20\du`, `100\du`, `140\du`,即三根为 `4\cos^210\du`, `4\cos^250\du`, `4\cos^270\du`,截图的式子就是这样来的。

此时 `(a - b)^2 (a - c)^2 (b - c)^2 = (p^2 - 4 q) q^2 - 2 p (2 p^2 - 9 q) r - 27 r^2` 其中 `p=a+b+c=6`, `q=ab+bc+ca=9`, `r=abc=3`,代入算出 `81`,所以 `(a-b)(a-c)(b-c)=9`。

123

Threads

463

Posts

3299

Credits

Credits
3299

Show all posts

 Author| TSC999 Posted at 2025-3-8 10:59:32
Last edited by hbghlyj at 2025-3-8 15:33:50此题使用拉格朗日乘子法比较容易理解,但是要先对目标函数取对数才方便求偏导数。只是先取对数这种做法是否有问题呢?
为方便求导,对原式取自然对数,并修正为
$$
\begin{aligned}
& F=\log (a-b)(a-c)(b-c) a b c+\lambda(a+b+c-6) \\
& =\log (a-b)+\log (a-c)+\log (b-c)+\log a+\log b+\log c+\lambda(a+b+c-6)
\end{aligned}
$$

依次对 $a, b, c$ 求偏导并令导数为 0 ,得(1),(2),(3)三等式
\begin{align}
& \frac{\partial F}{\partial a}=\frac{1}{a-b}+\frac{1}{a-c}+\frac{1}{a}+\lambda=0 \\
& \frac{\partial F}{\partial b}=\frac{-1}{a-b}+\frac{1}{b-c}+\frac{1}{b}+\lambda=0 \\
& \frac{\partial F}{\partial c}=\frac{-1}{a-c}+\frac{-1}{b-c}+\frac{1}{c}+\lambda=0
\end{align}
$(1)+(2)+(3):$
$\frac{1}{a-b}+\frac{1}{a-c}+\frac{1}{a}+\lambda+\frac{-1}{a-b}+\frac{1}{b-c}+\frac{1}{b}+\lambda+\frac{-1}{a-c}+\frac{-1}{b-c}+\frac{1}{c}+\lambda=0$
$\Rightarrow \frac{1}{a}+\frac{1}{b}+\frac{1}{c}=-3 \lambda$

$(1) \times a+(2) \times b+(3) \times c:$
$\frac{a}{a-b}+\frac{a}{a-c}+\frac{a}{a}+a \lambda+\frac{-b}{a-b}+\frac{b}{b-c}+\frac{b}{b}+b \lambda+\frac{-c}{a-c}+\frac{-c}{b-c}+\frac{c}{c}+c \lambda=0$
$\Rightarrow 6+a \lambda+b \lambda+c \lambda=0$
$\Rightarrow \lambda=-1$
\[
\tag4\therefore \frac{1}{a}+\frac{1}{b}+\frac{1}{c}=\frac{a b+b c+a c}{a b c}=3
\]
$(1) \times a^2+(2) \times b^2+(3) \times c^2:$
$\frac{a^2}{a-b}+\frac{a^2}{a-c}+\frac{a^2}{a}+a^2 \lambda+\frac{-b^2}{a-b}+\frac{b^2}{b-c}+\frac{b^2}{b}+b^2 \lambda+\frac{-c^2}{a-c}+\frac{-c^2}{b-c}+\frac{c^2}{c}+c^2 \lambda=0$
$\Rightarrow 3(a+b+c)+a^2 \lambda+b^2 \lambda+c^2 \lambda=0$
$\Rightarrow a^2+b^2+c^2=18$

$\because(a+b+c)^2-\left(a^2+b^2+c^2\right)=2(a b+b c+c a)=6^2-18=18$
$\Rightarrow a b+b c+a c=9$
代入(4)得 $a b c=3$
由此可知 $a, b, c$ 应为一元三次方程 $x^3+B x^2+C x+D=0$ 的三个根,由韦达定理可得
$$
\left\{\begin{array}{l}
a+b+c=6=-B \\
a b+b c+a c=9=C \Rightarrow x^3-6 x^2+9 x-3=0 \\
a b c=3=-D
\end{array}\right.
$$

解此方程可得
$$
\left\{\begin{array}{l}
a=2+2 \cos \frac{\pi}{9}=3.879385 \ldots \\
b=2-2 \sin \frac{\pi}{18}=1.652703 \ldots \\
c=2-2 \cos \frac{2 \pi}{9}=0.467911 \ldots
\end{array}\right.
$$

下面举一个具体例子说明:先取对数把目标函数加以改造似乎可行,但是只凭这一个成功的例子,并不是证明。如何证明或证否?
先取对数行不行.png

123

Threads

463

Posts

3299

Credits

Credits
3299

Show all posts

 Author| TSC999 Posted at 2025-3-9 14:05:47
用拉格朗日乘子法求极值,如果先对目标函数取对数后再求极值,是可以的。证明如下:
先取对数可行的证明.png

134

Threads

760

Posts

5439

Credits

Credits
5439

Show all posts

走走看看 Posted at 2025-3-13 10:29:15
Last edited by 走走看看 at 2025-3-13 10:54:28
kuing 发表于 2025-3-7 18:16
该截图是截自 artofproblemsolving.com/community/c6h432851p8689589 而不是知乎。

这其实是一 ...
Kung,我发现在电脑中未装抖音,也可观看抖音视频。

这个视频在   v.douyin.com/rzhhN8xY7I0/  这里。

701

Threads

110K

Posts

910K

Credits

Credits
94172
QQ

Show all posts

kuing Posted at 2025-3-13 13:10:59
Last edited by hbghlyj at 2025-4-9 10:50:44
走走看看 发表于 2025-3-13 10:29
Kung,我发现在电脑中未装抖音,也可观看抖音视频。

这个视频在   v.douyin.com/rzhhN8xY7I0/   ...

我这里点击还是提示要登录……

不要紧了,6# 已经写出来了,巧妙之处就在于如何解那个方程。

2

Threads

465

Posts

6357

Credits

Credits
6357
QQ

Show all posts

爪机专用 Posted at 2025-3-14 12:54:59
I am majia of kuing

134

Threads

760

Posts

5439

Credits

Credits
5439

Show all posts

走走看看 Posted at 2025-3-14 16:34:16
kuing 发表于 2025-3-13 13:10
我这里点击还是提示要登录……
对的,我的电脑也会出现要求登录的对话框。不过,把它关掉,不影响播放。

Comment

哦,确实可以。
那天我记得好像还没来得及关掉,就自动跳转到某个视频去了,不知啥原因,不管它了……  Posted at 2025-3-14 16:51
内容大概就是 6#
也可以看看 10# 的链接,也挺不错的  Posted at 2025-3-14 16:52

134

Threads

760

Posts

5439

Credits

Credits
5439

Show all posts

走走看看 Posted at 2025-3-14 17:03:13
TSC999 发表于 2025-3-8 10:59
此题使用拉格朗日乘子法比较容易理解,但是要先对目标函数取对数才方便求偏导数。只是先取对数这种做法是否 ...
没有什么纠结的啊。
取对数之前,先有一番分析。

虽然a确实可能小于b,也小于c,b也同时可能小于c,但此时(a-b)(a-c)(b-c)abc<0,当然<27,是没有问题的。

如果a-b、a-c都为负,可以调整为b-a、c-a,所以不影响最终结果。所以,讲解的老师说不妨设a>b>c>d=0。在此条件下,取对数肯定没有问题啊。

手机版Mobile version|Leisure Math Forum

2025-4-20 22:13 GMT+8

Powered by Discuz!

× Quick Reply To Top Return to the list